Tải bản đầy đủ (.doc) (64 trang)

Tập lồi và nón lồi

Bạn đang xem bản rút gọn của tài liệu. Xem và tải ngay bản đầy đủ của tài liệu tại đây (490.12 KB, 64 trang )

Chơng 1
Tập lồi và nón lồi
1. 1.Tập lồi
Cho X là không gian tuyến tính, Ă là tập các số thực.
1.1. 1.Định nghĩa. Tập A X đợc gọi là lồi nếu:
x1 , x2 A, Ă :0 1 x1 + (1 ) x2 A

.

1.1.2. Định nghĩa. Giả sử A X , x1 , x2 A . Đoạn nối x1 , x2 đợc định nghĩa
nh sau:
[ x1 , x2 ] = { x A : x = 1 x1 + 2 x2 ; 1 , 2 0, 1 + 2 = 1} .

Nhận xét : Tập A là lồi nếu: x1 , x2 A [ x1 , x2 ] A.
Ví dụ: Các đa giác lồi, đa diện lồi quen thuộc trong hình học sơ cấp 2 hoặc 3
chiều đều là các tập hợp lồi.
1.1.3. Định nghĩa. Véc tơ x X đợc gọi là tổ hợp lồi của các véc tơ:
n

n

i =1

i =1

i = 1sao cho x = i xi .

x1 , x2 ,..., xn nếu i 0 (i = 1, n),

1.1.4. Định lý. Tập A X là lồi khi và chỉ khi mọi véctơ:
x1 , x2 ,..., xm A , i 0 ,(i = 1, m) ;



m

m

= 1 , thi: x
i =1

i

i =1

i i

A.

Chứng minh
Ta chứng minh qui nạp theo m.
Điều kiện cần. Nếu A là tập lồi ta chứng minh:
m

m

i =1

i =1

i xi A , véc tơ, x1 , x2 , ..., xm A , i 0 ,(i = 1, m) : i = 1
Thật vậy:
2


m=2, theo định nghĩa 1.1 ta có: i xi A

(1)

i =1

Giả sử (1) đúng với m=k. Ta phải chứng minh (1) đúng đến m=k+1.
x1 , x2 ,..., xk , xk +1 A và i 0, (i = 1, k + 1) ,
k +1

k +1

=1
i =1

i

Nếu k +1 = 1 i = 0, = 1, k i xi = xk +1 A (theo giả thiết).
i =1

5


Nếu k +1 < 1 khi đó: 1 k +1 = 1 + 2 + K + k > 0 và

i
0, (i = 1, k ) .
1 k +1
k

i
i
=
1

y
=
xk A (theo giả thiết qui nạp).


1


1


i =1
i =1
k +1
k +1
k

Mặt khác

Vậy xk+ 1 A và y A nên (1 k +1 ) y + k +1 xk +1 A .
k +1
i
xk + k +1 xk +1 A hay i A .
i =1 1 k +1
i =1

k

(1 k +1 )

Theo nguyên lý qui nạp đpcm.
Điều kiện đủ. Cho m=2 ,theo định nghĩa 1.1.1 A là tập lồi .
Hệ quả. Giả sử A X là lồi , x1 , x2 ,..., xn A . Khi đó A chứa tất cả các tổ hợp
lồi của xi , (i = 1, n) .
1.1.5. Định lý. Giả sử A X ,( I ) là tập lồi, với I là tập chỉ số bất kỳ,
A cũng lồi .
Khi đó: A =
I
Chứng minh
x1 , x2 A x1 , x2 A , I
Do A lồi I cho nên: : 0 1 , x1 + (1 ) x2 A ( I )
x1 + (1 ) x2 I A x1 + (1 ) x2 A A là tập lồi (đpcm).
I

1.1.6. Định lý.
a) Giả sử A1 , A2 X là các tập lồi, R . Khi đó A1 + A2 ; A1 là các tập hợp
lồi.
b) A1 X 1 ; A2 X 2 thì tích đề các A1 ì A 2 lồi trong X 1 ì X 2 .
Chứng minh
Chứng minh A1 + A2 là tập lồi:`
x, y A1 + A2 x1 A1; x2 A2 : x = x1 + x2
y1 A1 , y2 A2 : y = y1 + y2 ;

6

va



1 , 2 0 : 1 + 2 = 1 , ta có: 1 x + 2 y = 1 ( x1 + x2 ) + 2 ( y1 + y2 )
= (1 x1 + 2 y1 ) + (1 x2 + 2 y2 ) .
Do A1, A2 là các tập lồi 1 x1 + 2 y1 A1 ; 1 x2 + 2 y2 A2 .
1 x + 2 y A1 + A2 A1 + A2 là tập lồi.
Chứng minh A1 là tập lồi :
x, y A1 x1 A1 : x = x1và y1 A1 : y = y1 , khi đó:

1 x + 2 y = 1 x1 + 2 y1 = (1 x1 + 2 y1 ) , 1 , 2 0 : 1 + 2 = 1
Do A1 là tập lồi 1 x1 + 2 x2 A1 ,x1 , y1 A1 và 1 , 2 0 : 1 + 2 = 1
1 x + 2 y A1 A1 là tập hợp lồi .
Chứng minh A1xA2 là tập lồi trong X1xX2.
z1 = ( x1 , y1 ) , ( x1 A1 , x1 A2 ) và z2 = ( x2 , y2 ) , ( x2 A1 , y2 A2 )
1,

2 0: 1+ 2=1, ta có: 1 z1 + 2 z2 = 1 ( x1 , y1 ) + 2 ( x2 , y2 )
= (1 x1 + 2 x2 , 1 y1 + 2 y2 ) .

Do A1, A2 lồi
1 x1 + 2 x2 A1 , 1 y1 + 2 y2 A2 1 z1 + 2 z2 A1 ì A2 .
Vậy A1xA2 là tập lồi.
1.1.7. Định lý. A, B là tập lồi, A X , B Y ; X , Y là các không gian tuyến
tính, T: X Y là toán tử tuyến tính. Khi đó:
a) T(A) lồi.
b) T-1(B) lồi.
Chứng minh
Chứng minh T(A) là tập lồi .
y1 , y2 T ( A) khi đó x1 , x2 A : y1 = T ( x1 ), y2 = T ( x2 )
1 , 2 0 : 1 + 2 = 1 ta có: 1 y1 + 2 y2 = 1T ( x1 ) + 2T ( x2 ) = T (1 x1 + 2 x2 )

Do A là tập lồi 1 x1 + 2 x2 A
Vậy T (1 x1 + 2 x2 ) T ( A) 1 y1 + 2 y2 T ( A) .

7


Vậy T(A) là tập lồi .
Chứng minh T-1(B) là lồi.
x1 , x2 T 1 ( B) y1, y2 B : x1 = T 1 ( y1 ) và x2 = T 1 ( y2 ) .
1 , 2 0 : 1 + 2 = 1 , ta có :

1 x1 + 2 x2 = 1T 1 ( y1 ) + 2T 1 ( y2 ) = T 1 (1 y1 + 2 y2 ) ,do B là lồi nên
1 y1 + 2 y2 B 1 y1 + 2 y2 T 1 ( B ) 1 x1 + 2 x2 T 1 ( B ) T 1 ( B ) là
lồi (đpcm).
1.1.8. Định nghĩa. Cho A X .Khi đó:
(i) Giao của tất cả các tập lồi chứa A đợc gọi là bao lồi của tập A, kí hiệu: coA .
(ii) Giao của tất cả các tập lồi đóng chứa A đợc gọi là bao lồi đóng của tập A,
kí hiệu: coA .
Nhận xét :
+ coA là một tập lồi. Đó là tập lồi nhỏ nhất chứa A.
+ A là tập lồi khi và chỉ khi A= coA
+ coA là một tập lồi đóng. Đó là tập lồi đóng nhỏ nhất chứa A.
+ A là tập lồi đóng khi và chỉ khi A= coA .
1.1.9. Định lý. Bao lồi đóng của tập A trùng với bao đóng của bao lồi A. Tức
là coA = coA
Chứng minh
Theo nhận xét của định nghĩa 1.1.8 thì coA là một tập hợp lồi chứa A coA
là tập lồi đóng chứa A.
Mặt khác coA là tập lồi đóng nhỏ nhất chứa A coA coA


(1)

Vì coA là tơng giao của tất cả các tập lồi (không đóng) chứa A coA coA
coA coA coA coA (vì coA là tập đóng).
Kết hợp (1) và (2) coA = coA .
1.1.10. Định lý. coA trùng với tất cả các tổ hợp lồi của A


Tức là: coA = x : x = i xi , i 0 : i =1 , x i A, I hữu hạn .
iI
iI



8

(2)


Chứng minh


Xét M = x : x = i xi , i 0 : i = 1, xi A, I - hữu hạn .

iI
iI

Lấy Ai là một tập hợp lồi bất kì chứa A.
Giả sử x M x = i xi , xi A
iI




xi A



A Ai xi Ai ,i I .

Mặt

khác Ai

lồi

nên

x Ai M Ai ,(i I ) M I Ai = coA hay M coA

(1)

iI

Ngợc lại x, y M và à1 , à2 0 : à1 + à2 = 1 ,ta có :
x, y M x = i xi , i 0 : i = 1, xi A , I1- hữu hạn .
iI1

iI1

y = j x j , j 0 : j = 1, y j A , I2-hữu hạn.

iI 2

jI 2

Xét: à1 x + à2 y = à1 i xi + à 2 j x j = (i à1 ) xi + ( j à 2 ) x j
iI1

jI 2

iI1

Rõ ràng xi A, (i I1 ) ; x j A, ( j I 2 ) và

jI 2

à + à
iI1

i

1

jI 2

j

2

= à1 + à 2 = 1 .


Vậy à1 x + à2 y M M là tập lồi.
Rõ ràng A M mà coA là tập lồi nhỏ nhất chứa A coA M
Từ (1) và (2) coA = M .
Hệ quả. Tập A lồi khi và chỉ khi A chứa tầt cả các tổ hợp lồi của A.
1.1.11.Định lý.Giả sử A X là một tập lồi .Khi đó:
Phần trong intA và bao đóng A của A là các tập hợp lồi.

(2)

Chứng minh
Chứng minh intA là tập lồi.
Lấy x1 int A, x2 A , khi đó tồn tại lân cận U của x1 sao cho
x = x1 + (1 ) x2 , 0 < 1 ta có: U + (1 ) x2 là lân cận của x
và U + (1 ) x2 A int A int A là tập lồi.
Chứng minh A là tập lồi.

9


Lấy x1 , x 2 A . Đặt x = x1 + (1 ) x2 , 0 < 1 .
Giả sử U là một lân cận lồi của O. Do x i A , nên xi+UI A , khi đó tồn tại
xi ' ( xi + U ) I A .Đặt x ' = x '1 + (1 ) x '2 , 0 < 1, ta có:
x ' ( x1 + U ) + (1 ) ( x2 + U ) = x + U , 0 < 1
x ' ( x + U ) I A hay ( x + U ) I A x A A là tập lồi .
1.2. Nón lồi
1.2.1. Định nghĩa
(i) Tập K X, đợc gọi là nón có đỉnh tại 0 nếu:
x K, > 0 ta có x K .
K đợc gọi là nón có đỉnh tại x0 nếu K x0 là nón có đỉnh tại 0.
(ii) Nón K có đỉnh tại 0 gọi là nón lồi nếu K là tâp lồi, có nghĩa là:

x , y K , , à > 0 x + ày K .
(iii) Giao của tất cả các nón lồi (có đỉnh tại 0) chứa tập A và điểm 0 là một nón
lồi và đợc gọi là nón lồi sinh bởi tập A, kí hiệu là KA.
Nhận xét:
+ Nếu K là nón thì -K cũng là nón và gọi là nón đối của K .
+ Một nón lồi cũng là nón nhng ngợc lại cha chắc đúng.
ví dụ: K = 1 + 2 , là nón nhng không phải là
nón lồi vì: 1+ 2 không phải là tập lồi.
+ KA=KcoA
1.2.2. Định lý. Tập K X là một nón lồi có
đỉnh tại 0 khi và chỉ khi
x, y K, > 0 x + y K, x K .
Chứng minh
Giả sử K là nón lồi , khi đó hiển nhiên: x K , > 0 , x > 0 .
1
1
x+ y K
2
2
Lại do K là nón lồi có đỉnh tại 0 nên với z K ta có x + y = 2 z K .
Do K là tập lồi ta có: x, y K thì z =

Ngợc lại:

10


x K , > 0 ta có x K .Vậy K là nón có đỉnh tại 0.
Ta chứng minh K là tập lồi .


(1)

Thật vậy: x, y K , 0 < < 1 , ta có (1 ) x, y K và (1 ) x + y K .
Vậy K là tập lồi .
(2)
Từ (1) và (2) K là nón lồi.
1.2.2.1. Hệ quả. Tập K X là một nón lồi khi và chỉ khi K chứa tất cả các
tổ hợp tuyến tính dơng của các phần tử của K .Tức là, nếu:
x1 , x2 ,..., xn K ; 1 , 2 ,..., n > 0 thì:

n

x K .
i =1

i i

1.2.2.2. Hệ quả. Giả sử A là tập bất kỳ trong X, K là tập tất cả các tổ hợp
tuyến tính dơng của A. Khi đó K là nón lồi nhỏ nhất chứa A.
1.2.3. Định lý. Giả sử K , ( I ) là các nón lồi (có đỉnh tại x 0) . Khi đó:
K , ( I ) là các nón lồi có đỉnh tại x0 .

Chứng minh
+ Giả sử K X , ( I ) là các tập lồi . Khi đó lấy
x1 , x2 K = I K x1 , x2 K , ( I ) x1 + (1 ) x2 K , ( I )
x1 + (1 ) x2 I K = K K = I K , ( I ) là tập lồi.

(1)

+ à 0, x K = I K x K , I à x I K = K


(2)

Từ (1) và (2) K = K là nón lồi .
1.2.4. Định lý.(Định lý Carathéodory đối với nón lồi ).Giả sử A Ă

n

sao cho

dimA=m, ( A ), KA là nón lồi sinh bởi tập A . Khi đó mỗi điểm x 0, x A
đều có thể biểu diễn dới dạng: x = 1 x1 + 2 x2 + ... + m xm , trong đó

i > 0 , xi A , (i = 1, m) , các điểm x1 , x2 ,..., xm độc lập tuyến tính.
Chứng minh
Lấy x K A , x 0 .Theo định lý ta có :
x = à1 x1 + à2 x2 + ... + à k xk , ( ài > 0 , xi 0 , i = 1, k )
Giả sử các véc tơ x1 , x2 ,..., xk , phụ thuộc tuyến tính, khi đó tồn tại các số

11

(1)


1, 2, , k không đồng thời bằng không sao cho:

1 x1 + 1 x2 + ... + k xk = 0 .

(2)


Nh vậy, trong các số 1, 2, , k có các i>0 (nếu không ta đổi dấu toàn bộ
1, 2, , , k).

{

}

Kí hiệu I = i = 1, k , i > 0 .Đặt = min(
iI

ài
) , ài ' = ài i , (i = 1, k ) , khi đó
i

à i> 0, (i = 1, k ) và có ít nhất một à i = 0 .
0

Mặt khác từ (1) và(2) ta có

k

k

k

k

à x = à x x = à x
i =1


'

i

i

i =1

i i

i =1

i i

i =1

i i

= x.

Nh vậy ta nhận đợc luôn biểu diễn của x dới dạng tổng không quá k-1 số hạng
khác không.
Lặp lại quá trình trên hữu hạn lần ta nhận đợc kết quả cần chứng minh.
1.2.5. Định lý. (Định lý Carathéodory đối với tập lồi ).
Giả sử A Ă n .Khi đó mỗi điểm của tập coA là tổ hợp lồi của không quá n+1
điểm khác nhau của A.
Chứng minh
Xét tập hợp B = { 1} ì A = { (1, x) : x A} Ă ì Ă n , ta có coB = { 1} ì coA .
Giả sử K B là nón lồi sinh bởi tập B , khi đó coB K B . Theo định lý 1.1.4
,nếu (1, x) coB thì tồn tại điểm (1, x1 ), (1, x2 ) , ...,(1, xm ) B và m số


1 , 2 ,..., m > 0 với m n + 1 : (1, x) = 1 (1, x1 ) + 2 (1, x2 ) + ... + m (1, xm ) B
x = 1 x1 + 2 x2 + ... + m xm

1 + 2 + ... + m = 1.

12


Chơng 2
hàm lồi và một số ứng dụng
Giả sử X là không gian lồi địa phơng, D X ,f : D Ă U { } .
2.1.Hàm lồi
2.1.1.Định nghĩa. Trên đồ thị hàm f, ký hiệu epi f, đợc định nghĩa:
epi f = { ( x, r ) D ì R : f ( x ) r} .
2.1.2. Định nghĩa
(i) Giả sử hàm f xác định trên tập D X . Khi đó hàm f lồi trên D nếu thoả
mãn các điều kiện sau:
x1 , x 2 D, 1 , 2 0 : 1 + 2 = 1 , thì:
f ( 1 x1 + 2 x 2 ) 1f (x1 ) + 2 f (x 2 ) .
(ii) Hàm 2 biến f(x, y) gọi là lồi trên miền xác định D f nếu Df là tập lồi và
(x1 , y1 );(x 2 , y 2 ) D f , 1 , 2 0 : 1 + 2 = 1 , thì:
f ( x1 + (1 )x 2 ; y1 + (1 )y 2 ) f (x1 , y1 ) + 2 f (x 2 , y 2 )
Tổng quát hàm n biến f ( x1 , x2 , ..., xn ) gọi là lồi trên Df nếu Df là tập lồi



(x1 , x 2 ,..., x n );(y1, y 2 ,..., y n ) D f , 1, 2 0 : 1 + 2 = 1 thì :
f ( 1 (x1 , x 2 ,..., x n ) + 2f (y1 , y 2 ,..., y n ) ) f 1f (x1 , x 2 ,..., x n ) + 2f (y1 , y2 ,..., yn )
ý nghĩa hình học:

f(x) là hàm lồi trên tâp D nếu
nh x1 , x 2 D thì cung AB
luôn nằm dới dây AB, với
A ( x1 ,f (x1 ) ) , B ( x 2 ,f (x 2 ) ) .

2.1.3. Định nghĩa
(i) Hàm f đợc gọi là lõm trên D nếu -f là lồi trên D . Tức là:

13


x1 , x 2 D, 1 , 2 0 : 1 + 2 = 1 : f ( 1 x1 + 2 x 2 ) 1f (x1 ) + 2 f (x 2 ) .
(ii) Hàm f đợc gọi là lồi chặt trên D
nêú: x1 , x 2 D, 1 , 2 > 0 , 1 + 2 = 1 thì:
f ( 1x1 + 2 x 2 ) < 1f (x1 ) + 2f (x 2 ) .
(iii) Hàm f đợc gọi là lõm chặt trên D nếu f là lồi chặt trên D .
Ví dụ:
1. Hàm số f(x)=x2 là lồi chặt trên Ă

.

Thật vậy, x1 , x 2 R, (x1 x 2 ) x1 , x 2 Ă , 1 , 2 > 0 : 1 + 2 = 1 ,
ta chứng minh: f ( 1 x1 + 2 x 2 ) < 1f (x1 ) + 2 f (x 2 )

(1)

(1) (1x1 + 2 x 2 ) 2 < (1 x12 + 2 x 2 ) 2
(12 1 )x12 + ( 2 2 2 )x 2 2 + 21 2 x1x 2 < 0 , (thay 2 = 1 1 )
1 (1 1)(x1 x 2 ) 2 < 0
Lại do: 1 > 0, (1 1 ) < 0 ,(x1 x 2 ) 1 (1 1)(x1 x 2 ) < 0

(1) đúng.

2. Hàm số y=a x+2 ; a, b Ă là lồi trên Ă .
Thật vậy: x1 , x 2 Ă , 1 , 2 0 : 1 + 2 = 1
f (1 x1 + 2 x2 ) = a(1 x1 + 2 x2 ) + b
= a (1 x1 + 2 x2 ) + b(1 + 2 )
= 1 ( ax1 + b) + (2 x2 + b)
= 1 f ( x1 ) + 2 f ( x2 )
f (x) = ax + b là hàm lồi trên Ă .
3. Hàm số f ( x, y ) = ax + by + c là hàm lồi trên Ă

2

Thật vậy : ( x1 , y1 ); ( x2 , y2 ) Ă 2 và 1 , 2 0 : 1 + 2 = 1 .Ta chứng minh:
f[ 1 ( x 1 , y1 )+ 2 ( x 2 , y 2 )] 1 f( x 1 , y1 ) + 2 f( x 2 , y 2 ).

(*)

a(1x1 + 2 x 2 )+b(1 y1 + 2 y2 )+c 1 (ax1 + by1 + c) + 2 (ax 2 + by2 + c )
c c (luôn đúng).

14


4. Hàm số f (x, y) = x 2 + y 2 là hàm lồi trên Ă 2 .
Thật vậy: ( x1 , y1 ); ( x2 , y2 ) Ă

2

và 1 , 2 0 : 1 + 2 = 1 .Ta chứng minh:


f[ 1 ( x 1 , y1 )+ 2 ( x 2 , y 2 )] 1 f( x 1 , y1 ) + 2 f( x 2 , y 2 ).

(1)

(1) (1x1 + 2 x 2 )2 + (1y1 + 2 y 2 ) 2 1 x12 + y12 + 2 x 2 2 + y 2 2
(1 x1 + 2 x 2 ) 2 + (1 y1 + 2 y 2 ) 2 12 (x12 + y12 ) + 2 2 (x 2 2 + y 2 2 ) +
21 2 (x12 + y12 )(x 2 2 + y 2 2 )
21 2 (x1 x 2 + y1 y 2 ) 2 1 2 (x 1 2 + y1 2 )(x 2 2 + y 2 2 ) ,

Do 1 2 0 x1y1+ x2y2 (x 1 2 + y 2 2 )(x 2 2 + y 2 2 )

(2)

Theo bất đẳng thức Bunhiacốpski
(x1 , y1 ); (x 2 , y 2 ) Ă 2 : (x12 + y12 )(x 2 2 + y 2 2 ) (x1 x1 + y1 y 2 ) 2 hay
2
2
2
(x 1 + y 2 )(x 2 + y 2 2 ) x1 x 2 + y1 y 2 (2) đúng .

Chứng minh hoàn toàn tơng tự ta có hàm: f (x) = x 21 + x 2 2 + ììì+ x 2 n là hàm
lồi trên Ă n , với: (x1 , x 2 ,..., x n ) Ă

n

.

Các tính chất cơ bản của hàm lồi
2.1.4. Định lý . Giả sử D X là tập lồi . Nếu fi, (i=1,, m) là các hàm lồi

m

trên D và 1 , 2 ,..., m Ă thì f ( x) = i f i là hàm lồi trên D.
+

i =1

Chứng minh
x1 , x2 D, 1 , 2 0 : 1 + 2 = 1
m

Ta có: f (1 x1 + 2 x2 ) = i f i (1 x1 + 2 x2 )
i =1

Vì fi (i = 1, m) là các hàm lồi trên D nên ta có:
f i (1 x1 + 2 x2 ) 1 f i ( x1 ) + 2 fi ( x2 ), i = 1,..., m .

15

(1)


Lại do i 0 nên ta có:

i fi (1 x1 + 2 x2 ) i 1 f i ( x1 ) + i 2 f i ( x2 ), i = 1,..., m
m


i =1


m

m

i =1

i =1



i f i (1 x1 + 2 x2 ) 1 i f i ( x1 ) + 2 i f i ( x2 )
m

i f i (1 x1 + 2 x2 ) 1 f ( x1 ) + 2 f ( x2 ) .
i =1

(2)

Từ (1) và (2) ta suy ra f (1 x1 + 2 x2 ) 1 f ( x1 ) + 2 f ( x2 ) (đpcm).
Ví dụ :các hàm f1 ( x) = ax+b, f 2 ( x) = x 2 là các hàm lồi trên R+ suy ra hàm
f ( x ) = 1 x 2 + 2 (ax + b) , (1 , 2 Ă ) là hàm lồi trên R+, hay các hàm bậc 2 là
lồi trên R+.
2.1.5. Định lý . Nếu f(x) là hàm số liên tục và lồi trên I(a, b) và nếu g(x) lồi và
đồng biến trên tập giá trị của g(x) thì gf(x) là lồi trên I(a, b) .
(với I(a, b) là ký hiệu của các đoạn, nửa đoạn, khoảng:[a, b] ; (a, b] ; [a, b) ; (a, b) ).
Chứng minh
Thật vậy, theo giả thiết f(x) là hàm số liên tục trên I(a, b) nên tập giá trị của
nó cũng là một tập có dạng I (c, d ) Ă .
Theo giả thiết f(x) là lồi trên I(a, b) x1 , x2 I (a, b) và
, 0 : + = 1 , ta có: f (x1 + x2 ) f ( x1 ) + f ( x2 ) .

Theo giả thiết g(x) là hàm số đồng biến nên ta nhận đợc :
gf ( x1 + x2 ) = g [ f ( x1 + x2 ) ] g [ f ( x1 ) + f ( x2 ) ] .

(1)

Do g(x) là hàm lồi nên:
g [ f ( x1 ) + f ( x2 ) ] g f ( x1 ) ] + g [ f ( x2 ) ] = gf ( x1 ) + gf ( x2 )

(2)

Từ (1) và(2) ta suy ra gf ( x1 + x2 ) gf ( x1 ) + gf ( x2 ) (đpcm).
Tơng tự ta cũng có các tính chất sau:
(i) Nếu f(x) là hàm số liên tục và lõm trên I(a, b) và nếu g(x) là lồi và nghịch
biến trên tập giá trị của f(x) thì gf(x) là hàm lồi trên I (a, b) .
(ii) Nếu f(x) là hàm số liên tục và lõm trên I(a, b) nếu g(x) là lõm và đồng
biến trên tập giá trị của f(x) thì gf(x) là hàm số lõm trên I(a, b) .

16


(iii) Nếu f(x) là hàm số liên tục và lồi trên I(a, b) và nếu g(x) là lõm và
nghịch biến trên tập giá trị của f(x) thì gf(x) là hàm lõm trên I(a, b) .
2.1.6. Định lý.Nếu f(x) là hàm số liên tục và đơn điệu (đồng biến hoặc nghịch
biến) trên I(a, b) và nếu g(x) là hàm ngợc của f(x) thì ta có các kết luận sau:
(i) f(x) lõm và đồng biến g(x) lồi và đồng biến.
(ii) f(x) lõm và nghịch biến g(x) lõm và nghịch biến.
(iii) f(x) lồi và nghịch biến g(x) lồi và nghịch biến.
Chứng minh
Suy trực tiếp từ hàm ngợc :Hàm ngợc luôn luôn cùng tính đơn điệu (đồng biến
hoặc nghịch biến ) với hàm xuất phát.

2.1.7. Định lý. Nếu f(x) là hàm số khả vi trên I(a, b) thì f(x) là hàm lồi trên
I(a,b) khi và chi khi f ' ( x) là hàm đơn điệu tăng trên I(a, b) .
Chứng minh
Giả sử f(x) là lồi trên I(a, b) khi đó x1 < x < x2 , ( x, x1 , x2 I (a, b)) ta có :
x2 x
x-x1
x x x x1
>0 ;
>0 ; 2
+
= 1 và
x2 x1
x 2 -x1
x2 x1 x2 x1
f(

x1 + x2
f ( x1 ) + f ( x2 )
)
2
2
Chọn x =

x -x
x x1
x1 + x2
f ( x1 ) +
f ( x2 ) suy ra
ta thu đợc f ( x) 2
x

-x
x

x
2
2
1
2
1

f ( x) f ( x1 ) f ( x2 ) f ( x1 )

x x1
x2 x1
Từ (1) và (2) suy ra

(1) và

f ( x) f ( x2 ) f ( x2 ) f ( x1 )

(2)
x x2
x2 x1

f ( x) f ( x1 ) f ( x) f ( x2 )

.
x x1
x x2


Trong (1) cho x x1 ta thu đợc: f ' ( x1 )

f(x 2 ) - f(x1 )
x2 x1

Tơng tự trong (2) cho x x2 ta thu đợc: f ' ( x2 )

17

(*)

f(x 2 )-f(x1 )
x2 x1

(3)
(4)


Từ (3) và (4) ta nhận đợc f ' ( x1 ) f ' ( x2 ) tức hàm số f(x) là hàm đơn điệu
tăng.
Ngợc lại: Giả sử f(x) là hàm đơn điệu tăng và
x1 < x < x2 ; ( x, x1 , x2 I (a, b) ) .
Theo định lý Lagrănge x 3 , x 4 , với x1 < x3 < x < x4 < x2 sao cho
f ( x) f ( x1 )
f ( x2 ) f ( x )
= f ' ( x3 ) ,
= f ' ( x4 ) . Do f ' ( x3 ) f ' ( x4 ) nên
x x1
x2 x
f ( x) f ( x1 ) f ( x2 ) f ( x)


, tức là ta có (*) (đpcm).
x x1
x2 x
2.1.8. Định lý . Nếu f(x) khả vi bậc 2 trên I(a, b) thì f(x) là lồi ( hoặc lõm)
trên I(a, b) khi và chỉ khi f '' ( x ) 0 ,

( f (x) 0 )
''

trên I (a, b).

Chứng minh
Nếu f(x) lồi trên I(a, b) thì ta chứng minh f '' ( x) 0 .
Thật vậy, theo định lý 2.1.7 thì f ' ( x) là hàm đơn điệu tăng nên
x1 < x ; ( x, x1 I (a, b) ) thì f ' ( x1 ) f ' (x) , nên ta có:
f ' ( x) f ' ( x1 )
f ( x) = lim
0.
x x
x x1
''

1

Do x1 bất kỳ : x1 < x x I(a, b) thì f '' ( x) 0 .
Ngợc lại: f '' ( x) 0 x I ( a, b) thì x1 < x < x2 , ta có:
f ' ( x) f ' ( x1 )
'
'

f ( x1 ) = lim

0

f
(
x
)

f
( x1 )
x x
x x1

(1)

f ' ( x2 ) f ' ( x )
'
'
f ( x2 ) = lim

0

f
(
x
)

f
(x )

2
x x
x2 x

(2)

''

1

''

1

Từ (1) và (2) f ' ( x1 ) f ' (x) f ' (x 2 ) f ' ( x1 ) f ' ( x2 ) nên f ' ( x) là hàm
đơn điệu tăng. Theo định lý 2.1.7 ta suy ra f(x) là hàm lồi trên I(a, b).
2.1.9.Định lý. Nếu f(x) là lồi trên I(a, b) thì tồn tại các đạo hàm một phía
f ' ( x) , f + ' (x) , x (a,b) và f ' ( x) f + ' (x) .

18


Chứng minh
x0 [a, b] , x0 cố định, chọn các số dơng tùy ý u, v sao cho
( x0 u ) (a, b), ( x 0 + v) (a,b) . Khi đó theo (*) ở định lý 2.1.7 ta có
f ( x 0 ) f ( x0 u ) f ( x 0 + v ) f ( x0 )
<
u
v


(1)

chọn v>v để ( x0 + v ' ) (a, b) thì x0 < x0 + v < x0 + v ' và theo (1) thì
f (x 0 + v) f (x 0 ) f (x 0 + v ' ) f (x 0 + v)

v
v' v

(2)

'
Biến đổi (2) ta thu đợc: f ( x0 + v) f ( x0 ) f ( x0 + v )' f ( x0 )
(3)
v
v
f ( x 0 + v) f ( x 0 )
Hệ thức (3) chứng tỏ hàm số g(v) =
là một hàm đơn điệu
v
tăng và khi v giảm dần tới 0 thì g(v) đơn điệu giảm và bị chặn (theo (1) ) nên

tồn tại

lim+ g (v) = lim+

v 0

v 0

f ( x0 + v) f ( x0 )

= f ' ( x0 ) .
v
+

'
Tơng tự ta cũng có f ( x0 ) .


Nhận xét rằng nếu trong (1) cho u , v 0+ thì ta thu đợc bất đẳng thức:
f ' ( x0 ) f ' ( x0 )


+

(đpcm).

2.1.10. Định lý. Nếu f(x) là hàm lồi trên I(a, b) thì f(x) liên tục trên I(a, b).
Chứng minh
'
'
Theo định lý 2.1.9 thì f ( x); f ( x ), x (a,b) f ( x) vừa liên tục trái


+

vừa liên tục phải f ( x) liên tục x (a, b) .
Nhận xét: Hàm lồi trên [a;b] có thể không liên tục tại đầu mút của [a;b].
x 2 x khi x (0,1)
Thật vậy, hàm số: f ( x ) =
khi x = 1

1
là hàm số lồi trên [0, 1] nhng không liên tục tại x=1.
2.1.11. Định lý. (Mối quan hệ giữa hàm lồi và tập lồi).
Hàm f(x) là lồi trên D khi và chỉ khi epi f là tập lồi trong X ì Ă .
Chứng minh

19


a) Giả sử f(x) là hàm lồi .
Xét ( x1 , r1 ) epi f ; ( x2 , r2 ) epi f ; 1 , 2 > 0 , 1 + 2 = 1.
Theo định nghĩa epi f ta có:
x D
( x1 , r1 ) epi f 1
f ( x1 ) r1
x D
( x2 , r2 ) epi f 2
f ( x2 ) r 2
Vì D là tập lồi 1x1 + 2x 2 D
Do f lồi nên f (1 x1 + 2 x2 ) 1 f ( x1 ) + 2 f ( x2 ) 1r1 + 2r2
Chứng tỏ rằng
(1 x1 + 2 x2 ; 1r1 + 2 r2 ) epi f 1 ( x1 , r1 ) + 2 ( x2 , r2 ) epi f
Vậy epi f là tập lồi trong X ì Ă .
b)Ngợc lại: Giả sử epi f là tập lồi.Ta sẽ chứng minh f là hàm lồi.
Giả sử f không phải là hàm lồi khi đó
x1 , x2 D, x1 x2 , à1 0, à2 0 : à1 + à 2 = 1 , sao cho:
f ( à1 x1 + à2 x2 ) à1 f ( x1 ) + à 2 f ( x2 )

(1)


Ta có: ( x1, f ( x1 ) ) epi f , ( x2, f ( x2 ) ) epi f .Do epi f là tập lồi nên

à1 ( x1, f ( x1 ) ) + à 2 ( x2, f ( x2 ) ) epi f


( à1 x1 + à2 x2 , à1 f ( x1 ) + à2 f ( x2 ) ) epi

f

Theo định nghĩa epi f ta có: f ( à1 x1 + à 2 x2 ) à1 f ( x1 ) + à 2 f ( x2 )

(2)

Ta thấy (1) và (2) mâu thuẫn.Vậy f là hàm lồi.
Biểu diễn của hàm lồi và hàm lõm
Ta thấy f(x) lồi trên I (a, b) f ''( x) 0, x I ( a, b) , từ đó ta có nhận xét:
Khi hàm f lồi trên I(a, b) thì đạo hàm bậc nhất là một hàm đơn điệu tăng,
do vậy ta có tính chất:
2.1.12. Định lý. Hàm f(x) lồi trên I(a, b) khi và chỉ khi tồn tại hàm g(x) đơn


điệu tăng trong I(a, b) và số c (a, b) sao cho: f ( x) = f (c) + g (t )dt .
0

20


2.1.13. Định lý. Nếu f(x) lồi trên I(a, b) thì mọi cặp x0 , x I (a, b) , ta đều có:
f ( x) f ( x0 ) + f '( x0 )( x x0 ) .


(1)

Chứng minh
(1) f '( x 0 )
Và f '( x 0 )

f ( x) f ( x0 )
, khi x > x0
x x0

, với x0 , x I (a, b)

(2)

f ( x) f ( x0 )
, khi x < x0
x x0

, với x, x0 I (a, b)

(3)

Theo định lý Lagrange và do f là hàm lồi thì :
f '( x 0 ) = lim

f ( x) f ( x0 ) f(x)-f(x 0 )

, khi x > x0
x x0
x-x 0


f ' ( x 0 ) = lim
x x

f ( x) f ( x0 ) f(x) - f(x 0 )

, khi x < x0
x x0
x - x0

x x0

0

Suy ra (2) và (3) đúng (đcpm).
Dễ thấy (1) xảy ra = khi và chỉ khi x = x 0.Vậy (1) có thể phát biểu dới dạng:
f ( x) = umin
( f (u ) + f ' (u )( x u )) .
I ( a ,b )
Tơng tự ta cũng có biểu diễn đối với hàm lõm .
Nếu f(x) lõm trên I(a, b) thì mọi cặp x0 , x I (a, b) , ta đều có:
f ( x) f ( x0 ) + f ' ( x0 )( x x0 ) .

(4)

Dễ thấy (4) xảy ra = khi và chỉ khi x = x0.Vậy (4) có thể phát biểu dới dạng:
f ( x) = max ( f (u ) + f ' (u )( x u )) .
uI ( a ,b )

Tơng tự ta cũng có biểu diễn đối với lớp hàm lồi và lớp hàm lõm nhiều biến.

Xét hàm số thực nhiều biến F(x1, x2, , xn).Giả sử ứng với mọi bộ số
(z1, z2, , zn) mà z1 z 2 ... z n ta đều có:
n

F ( x1 , x2 ,.., xn ) F ( z1 , z 2 ,.., z n ) + ( xi zi )
i =1

F
zi

n

F
Khi đó hiển nhiên: F ( x1 , x2 ,.., xn ) = max F ( z1 , z 2 ,.., z n ) + ( xi zi ) .
zi
i =1


21


2.2.Một số ứng dụng của hàm lồi
2.2.1. Bất đẳng thức với hàm lồi
2.2.1.1. Bất đẳng thức Jen sen
Cho f(x) là lồi trên [a;b] và giả sử x1, x2, , xn [a;b], và i >0, i = 1, n ,
n
n

1 + 2 +...+ n = 1 . Chứng minh rằng: f i x i ữ i f(x i )
i =1

i =1
-1 .Ta chứng minh (1) đúng đến n.Chứng minh
Ta chứng minh qui nạp theo n.
Với n=2, thì (1) đúng (theo định nghĩa hàm lồi).
Giả sử (1) đúng đến n
n

Thật vậy , ta có:

x
i

i=1

i

= 1x1 + 2 x2 + ... + n 2 xn 2 + n1 xn1 + n xn .

(1)

(2)

n-2

Đặt = i , khi đó 0< <1 và từ (2) ta có thể viết:
i=1

n-2

n


n-1

x = + (1- ) ( 1- x
i

i=1

i

i

i=1

n-1

+

n
xn) .
1-

n-1 n


+
=1 x*= n-1 xn-1+ n xn [xn-1;xn] [a;b].
1- 1-
1-
1-

áp dụng giả thiết qui nạp với n-1 điểm x1, x2, , xn-2 và x*, ta có :

Do

n2
n

n-2

*
f i x i ữ )= f i xi ữ+ (1- )x ) i f(x i ) + (1- )f(x*).
i =1

i=1

i =1
Lại theo định nghĩa hàm lồi ta có:




f(x*)=f( n-1 xn-1+ n xn) n-1 f(xn-1) + n f(xn)
1-
1-
1-
1-

n
n
Thay (4) vào (3) ta có : f i x i ữ i f(x i ) .

i =1
i =1
Vậy bất đẳng thức đúng đến n .
1
Nhận xét: Nếu 1 = 2 = ... = n = thì ta có:
n

x +x +...+x n
f 1 2
n

A. Các hệ quả

1
ữ [ f ( x1 ) + f ( x2 ) + ... + f ( xn ) ] .
n

22

(3)

(4)


HÖ qu¶1. BÊt ®¼ng thøc Cauchy
a +a +...+a n
≥ n a1a 2 ...a n
Cho a1, a2, …, an ≥ 0. Khi ®ã 1 2
n
Chøng minh

XÐt hµm sè y = − ln x , víi x > 0 ta cã f '' ( x) =

1
> 0 ⇒ f ( x) lµ hµm låi khi
x2

x > 0 , theo bÊt ®¼ng thøc Jensen ta cã :
 a + a + ... + an  1
f 1 2
÷ ≤ ( f (a1 ) + f (a2 ) + ... + f (an ) )
n

 n
1
 a + a + ... + an 
⇔ − ln  1 2


( ln(a1 ) + ln(a2 ) + ... + ln(an ) )
÷
n
n


 a + a + ... + an 
⇔ ln  1 2
÷ ≥ ln
n




(

n

)

a1a2 ...an .

Do y = ln( x) lµ hµm ®ång biÕn ∀ x > 0 ⇒

a1 + a2 + ... + an n
≥ a1.a2 .....an .
n

HÖ qu¶2. BÊt ®ẳng thøc Bunhiac«pski
Cho 2n sè thùc a1, a2, …, an vµ b1, b2, …, bn khi ®ã
n

n

n

( ∑ a i )( ∑ b i ) ≥ ( ∑ a i bi )2.
2

i =1

i=1


2

i=1

Chøng minh
XÐt hµm sè f(x)=x2 ⇒ f”(x)=2>0 ⇒ f(x) lµ hµm låi trªn ¡ .
Theo bÊt ®¼ng thøc Jensen ta cã: ∀ α i ≥ 0 , i = 1, n vµ

n

∑ α =1, ta cã:
i

i=1



 α
 n α
α2
αn
1
f n
x1 + n
x 2 + ×××+ n x n  ≤ ∑ n k f ( xk )
 α
αi
αi  k=1 ∑ α i




i
 i=1

i=1
i=1
i=1

23


2



1



x12 + n 2 x2 2 + ììì+ n n xn 2
2
n
n
1
n x1 + n x 2 + ììì+ n x n
i
i




i


i=1
i=1
i=1 i i=1
i=1 i i= i
(1 x1 + 2 x2 + ... + n xn ) 2 (1 x12 + 2 x2 2 + ììì+ n xn 2 )(1 + 2 + ììì+ n ) .(*)

Nếu tồn tại bk = 0 thì ta bỏ đi cặp (ak, bk) và cứ nh vậy cho đến khi chỉ còn
cặp (ai, bi) mà bi 0. Vậy ta có thể giả sử bk 0 , k = 1, n .
Thay i =bi 2 ; i xi 2 = ai 2 xi 2 =

ai 2
ai

x
=
i
bi 2
bi

vào (*) ta suy ra điều phải

chứng minh.
Hệ quả3. Bất đẳng thức Holder
1

1


Mọi bộ số không âm (xk), (yk), k = 1, n và mọi cặp số p , q 0: p + q =1 ta đều có
n

x

k=1

1
p

k

1
q

p
q
y k
x k ữ x k ữ .
k=1
k=1

n

n

Chứng minh
Do p , q > 0 và

1 1

+ =1 p>1 . Xét hàm f ( x) = x p , với x>0 ta có
p q

f ' ( x) = px p 1 , f '' ( x) = p ( p 1) x p 2 > 0 , x > 0
Vậy f(x) là hàm lồi khi x>0, theo bất đẳng thức Jensen ta có:
n
n
f i z i ữ i f (z i ) .
i=1
i=1
Với zk = xk yk1q , k = 1, n ,

i =

(1)
yi q

n

n

y

q
k

ta có: xi > 0 , i > 0 i = 1, n và i = 1
i=1

k=1


24


n

Do

yk q

n

∑ α k zk = ∑
k=1

k=1

n

∑y

q

n

1

xk yk 1−q =

n


∑y

j

j=1

∑x y
k

q

k=1

j

k

.

(2)

j=1

Thay (2) vµo (1)


 1
 n
 yq

j
∑
 j=1
Do

p



n
x k yk  ≤


k =1



y qk

n


k=1

n

∑y
j=1

q

j

x pk y pk (1-p)

(3)

1 1
+ = 1 ⇒ p +q - pq = 0
p q

⇒ y qk y p(1-q)
= y qk + p - p q =y 0k = 1 .
k

(4)

Tõ (3) vµ (4)

p

 n

x
y
∑ k k ÷
 ≤
⇒  k =1
p
 n q
 ∑y j ÷

 j=1 

n

∑x

q
k

∑y

q
j

k =1
n

p −1

p

 n

 n p  n q
⇔  ∑x k y k ÷ ≤  ∑x k ÷  ∑y j ÷ .
 k =1

 k =1   j =1 

j =1


1
p

p −1
p



 
⇔ ∑x k y k ≤  ∑x pk ÷  ∑y qj ÷
k=1
 k =1   j =1 
n

n

1
p

n

1
q



 
⇔ ∑ x k y k ≤  ∑ x pk ÷  ∑ y qj ÷ .
k =1

 k =1   j=1 
n

n

n

-Tõ bÊt ®¼ng thøc Holder ta thu ®îc bÊt ®¼ng thøc Mincowski
BÊt ®¼ng thøc Mincowski 1

25


Mọi bộ số không âm (xk), (yk), k = 1, n và số dơng p>1, ta đều có:
1

1

1

n
n p p n p p .
pp
( x k + yk ) x k ữ + y k ữ
k =1

k =1
k =1

(1)


Chứng minh
p

n

( x k + yk ) =

Ta có:

k =1

n

x k ( x k + yk )

p 1

n

+

k =1

y ( x
k

k

k =1


+ yk )

p 1

(2)

Sử dụng bất đẳng thức Holder đối với từng số hạng ở vế phải của (2) ta thu đợc:
p

n

( x
k =1

k

1
p

(



+ y k ) x pk ữ ( x k + y k )
k =1 k =1
n

n


1
p

)

p 1 q





1
q

(



+ y pk ữ ( x k + y k )
k =1 k =1
n

n

( xk + y k )
k =1

q=

p


n

)

p 1 q

1
q

.



1
1
1
n
n
n
q
p
p




p 1
q
x pk ữ + y kp ữ ( xk + y k )

thay
k =1
k =1 k =1




(

)

p
p 1
1
p

1

1

n
n
p
p
Ta đuợc ( x + y ) x p + y p
k k
k ữ
k ữ
k =1
k =1


k =1
n

Với

p

1 1
+ =1
p q

- Từ bất đẳng thức Holder, ta có kết quả

26

(đpcm).


Với mọi bộ số không âm (xk),và số p > 1 ta đều có thể biểu diễn dạng tuyến

tính sau

1
p

n

x k yk .


x ữ = max
R(y)
k=1
k =1
n

p
k

Trong đó R(y) là tập hợp đợc xác định nh sau:
n


1
1
R ( y ) = ( y1 0, y 2 0, ììì, y n 0 ) : y qk = 1, + = 1 .
p
q
k =1


Bất đẳng thức Mincowski 2.

Với mọi bộ số không âm (xk) ,(yk), ( k = 1, n ) và số p (0, 1), ta đều có:
n

( x
k =1

k


1
p

1
p





+ y k ) x kp ữ + y pk ữ .
k =1
k =1
n

1
p

n

(1)

Chứng minh
1

1

Đặt x =u p ,
y k = v kp thì (1) có dạng :

k
k

n
u
kữ

k =1


Vì 0 < p < 1 nên

1
p


1 p
n 1



+ v k ữ u kp + v kp ữ .


k =1
k =1


1
p


n

1
> 1 và theo bất đẳng thức Mincowski 1 với p >1 ta có
p
p

1
1
n

n
n
n
p
p
u + v = max t u + t v .
kữ
1 k ữ 2 k ữ
k ữ
R (t)

k =1

k =1
k =1
k =1




Trong đó R(t) :=

{( t

1

0, t 2 0 ) , mà t1q + t 2p =1, p+q=1}

n

n

Mà, max t1 u k ữ + t 2 v k ữ
R (t)
k =1
k =1

n

max ( t u

1 k

+ t2 vk )

k =1

p


1
1p

Và max(t1u k + t 2 v k ) = u k + v kp ữ n ên ta dễ dàng thu đ ợc (2).


k =1
k =1

Hệ quả 4. Bất đẳng thức Petrovica
n

n

27

(2)


Cho hàm f(x) là hàm lồi trên [ 0, a]. Giả sử xi [0, a],( i = 1, n ) sao cho
n

x i [ 0,a ] . Khi đó
i =1

n
f
x

f

(
)

i
x i ữ + ( n - 1) .f ( 0 ) .
i=1
i=1
Chứng minh
n

n

Do

n

x [ 0,a ] và 0 [ 0,a ] ,
i =1

i

x

j

x x

i

xi


+

n

i

i =1

j i
n

i =1

= 1 nên theo định nghĩa hàm lồi thì

n


xj

xj
x

n
x i n ji
ji
i
f (x i ) = f n
xi + n

. 0 n
f x i ữ + n .f(0) , (i = 1,n). (1)

x i =1

xi
x i i =1 x i


i


i =1
i =1
i=1
i =1
Cộng từng vế của n bất đẳng thức dạng (1) ta đợc:

x
jữ
xi


f (x i ) n
f x i ữ + f(0). jn i ữ.


i =1
i =1
i =1

x i i =1

xi ữ
i =1
i =1
n

n

n

n

n
Suy ra: f (x i ) f x i ữ + ( n - 1 ) f (0) .
i =1
i =1
n

Chú ý: Ta cũng có một suy rộng của bất đẳng thức Petrovica.
Cho f (x) là hàm lồi trên [0, a] với a > 0 và p1,..., pn là các số không âm
sao cho:


x i 0,



p i x i , i = 1,n và


i =1

n

n

p x
i

i

< a.

i =1

n
n

Khi đó ta có: pi f (x i ) f p i x i ữ + p i - 1 ữ f (0).
i =1
i =1
i =1

B. áp dụng bất đẳng thức Jensen và các bất đẳng thức hệ quả vào trong
n

mộtsố bất đẳng thức đại số khác
Bài toán 1. Cho n số dơng x1, x2, ..., xn , các số:

ma =


x1 + x 2 + ... + x n
; m g = n x1x 2 ...x n
n
28


x12 + x 22 + ...+ x 2n
n
mq =
; mh =
1 1
1
n
+ + ...+
x1 x 2
xn
Tơng ứng gọi là trung bình cộng, trung bình nhân, trung bình toàn phơng,
trung bình điều hòa của n số đã cho chứng minh rằng:

mh mg ma mq.
Chứng minh
Xét hàm số: f(x) = x2, x > 0 ta có f (x) = 2 > 0 , x > 0
1
x1 + x 2 + ...+x n

( f (x1 ) + ...+f(x n ) )
f(x) lồi trên (0,+ ) f

n

n


2

2
2
2
x
+
x
+
ì
ì
ì
+
x
x
+
x
+
...+x


1
2
n
1
2
n



n
n



x + x 2 +...+ x n
hay 1

n



x12 + x 22 + ...+ x 2n m m
a
q
n

(1)

Dấu = xảy ra tại (1) x1 = x 2 =...= x n
Xét hàm số: f(x)=-ln(x), x > 0 ta có :

f '' (x) =

1
> 0 f(x) = - ln(x) là hàm lồi trên (0,+ ), áp dụng bất
x2


đẳng thức Jensen ta có:
1
1
1
+
+ ... +
1 1
1
1
x
x2
xn
ln 1
- ln + ln +...+ ln ữ
n
n x1
x2
xn
ln

n
1
1
1
+
+ ...+
x1
x2
xn






1
ln

x1x 2 ...x n

29

1
n


Tài liệu bạn tìm kiếm đã sẵn sàng tải về

Tải bản đầy đủ ngay
×